The graphs below have the same shape. What is the equation of the red graph? g(x) = A. g(x)=2^x+3-2 B. 2^x-3-2 C. 2^x-2-3 D. g(x)=^x+2-3

Answers

Answer 1

The equation of the red graph is g(x)=^x+2-3.

Since the upscaling in the y-axis is not the main focus of this equation (being g(x) and not g(y)), the change is not applied to the x-axis.

What is the equation of the graph?

The y-intercept and the slope in order to write the equation in y-intercept (y=mx+b) form. The slope is the change in y over the change in x.

Hence, we can rule out (-3)^2 and (x+3)^2 since they apply changes to the x-axis.

Finally, since this is an upscaling of the y-value on the y-axis, the answer is x^2 + 3 and not x^2 - 3.

To learn more about graphs visit:

https://brainly.com/question/4025726

#SPJ1


Related Questions

find the indicated side of the right triangle

Answers

[tex]\quad \huge \quad \quad \boxed{ \tt \:Answer }[/tex]

[tex]\qquad \tt \rightarrow \:x = 3\sqrt3 [/tex]

____________________________________

[tex] \large \tt Solution \: : [/tex]

[tex]\qquad \tt \rightarrow \: \tan(60) = \cfrac{x}{3} [/tex]

[tex]\qquad \tt \rightarrow \: \sqrt{3} = \dfrac{x}{3} [/tex]

[tex]\qquad \tt \rightarrow \: x = 3 \sqrt{3} [/tex]

Answered by : ❝ AǫᴜᴀWɪᴢ ❞

The price of 11 pizzas and 5 hamburgers is 40$. The price of 1
hamburger is 2.50$. What is the price of a hamburger?

Answers

Step-by-step explanation:

you mean, what is the price of a pizza, right ? and I guess it is pizza slices ...

1 hamburger = $2.50

5 hamburgers = 5×1 hamburger = 5×2.5 = $12.50

that leaves for the 11 pizzas (slices)

40 - 12.5 = $27.50

so, 1 pizza (slice) is

27.5 / 11 = $2.50

Answer:

One hamburger costs $2.50

One pizza also costs $2.50

Step-by-step explanation:

Let's say Nigel is the one purchasing lots of food for no reason.

Nigel bought 11 pizzas and 5 hamburgers for $40 (DAMMMM WHAT COUPON IS THAT??)

We can write this algebraically:

11p + 5b = 40

We know that the price of 1 Hamburger is $2.50.

Now we can plug that into out equation and find the price of one pizza:

11p + 5(2.50) = 40

11p + 12.5 = 40

11p = 27.5

p = 2.5

Now, we have the price of one pizza, $2.50

What is one possible value of 2x

Answers

Answer:

really good

Step-by-step explanation:

thanks for your help you with that do not have a copy of the receipt for your time to help you with the

Which of the following could be the first step in solving the equation below 4+2log_(3)x=17

Answers

Answer:

subract 4 from both sides

Step-by-step explanation:


Two trains going in opposite directions leave at the same time. Train B travels 5 mph faster than train A. In 3 hours the trains are 315 miles apart. Find the speed of each train

Answers

Answer:

Step-by-step explanation:

Graph the line passing through (6, 1) whose slope is m = 3.

Answers

Answer: [tex]y=3x-17[/tex]

Step-by-step explanation:

use point-slope form:

[tex]y_{1} -y=m(x_{1} -x)[/tex]

[tex]y-1=3(x-6)[/tex]

[tex]y-1=3x-18[/tex]

[tex]y=3x-17[/tex]

Answer:

equation: y = 3x - 17

[image attached]

Step-by-step explanation:

we know that the equation for a line is y = mx + b

we are given a point, (6, 1)

remember that a point is (x, y)

we can plug these sample x and y values into our equation, replacing x and y

 y = mx + b

 1 = m6 + b

we know that m = 3 [this was a given], so we can plug this into our equation

1 = (3)(6) + b

1 = (18) + b

1 = 18 + b       [we can now solve for "b"]

1 = 18 + b

-18    - 18

-17 = b

so, we can re-write our equation so that we can graph it:

y = 3x - 17

we can now plug in x-values to graph this:

(you'll see these written in the table on the graph)

y = 3(0) - 17

y = -17

y= 3(1) - 17

y = 3 - 17

y = -14

y = 3(2) - 17

y = 6 - 17

y = -11

y = 3(3) - 17

y = 9 - 17

y = -8

y = 3(4) - 17

y = 12 - 17

y = -5

y = 3(-1) - 17

y = -3 - 17

y = -20

y = 3(-2) - 17

y = -6 - 17

y = -23

y = 3(-3) - 17

y = -9 - 17

y = -26

[there is an image attached of my graph]

hope this helps!! :)

A shopkeeper buys 72 articles for $82.80. How much will he have to pay if he buys 150 such articles.​

Answers

Answer: He have to pay $172.5 if he buys 150 such articles.

72 articles for --> $82.80
1 article for - - - > 82.80/72

150 articles for --> (82.80/72)*150
                           = $172.5

Answer:

$172.5

Step-by-step explanation:

Given:

Shopkeeper buys = 72 articles

Price = $ 82.8

To Find:

The amount he'll need to pay If he buys 150 articles.

Solution:

Amount if he buys 150 articles

[tex] \boxed{\rm \: No. \: of \: articles \: he \:will \: buy × \cfrac{Cost \: of \: that \: article \: he \: has}{Number \: of \: articles \: he \: has}} [/tex]

[tex] = \$ \bigg(\rm150 * \cfrac{82.80}{72}\bigg) [/tex]

[tex]= \$ (150* 1.15)[/tex]

[tex] = \$172.5[/tex]

So we can conclude that:

$172.5 he will have to pay if he buys 150 such articles.

Solve the equation.
-x+8+3x = x-6
O x=-18
O x=-14
O x = 2
O x = 4

Answers

Answer:

x = -14

Step-by-step explanation:

- x + 8 + 3x = x - 6

- x + 8 + 3x - x + 6 = 0

x + 14 = 0

x = -14

Which statements about the system are true? Select two options.
y=-x-4
3y - x = -7
O The system has one solution.
O The system consists of parallel lines.
O Both lines have the same slope.
Both lines have the same y-intercept.
The equations represent the same line.

Answers

The system of equation has one solution

How to determine the true statements?

The equations are given as:

y = -x - 4

3y -x = -7

Rewrite the first equation as:

y + x = -4

Add y + x = -4 to the second equation to eliminate x

4y = -11

Divide by 4

y = -11/4

Substitute y = -11/4 in y + x = -4

-11/4 + x = -4

Make x the subject

x = -4 + 11/4

Evaluate

x = -5/4

The above means that the system of equation has one solution

Read more about system of equations at:

https://brainly.com/question/14323743

#SPJ1

Option A and B. The system has one solution and the system consists of parallel lines.

Slope of the lines

The slope of the lines is calculated as follows;

y = -x - 4

slope = - 1

3y - x = -7

3y = x - 7

y = x/3 - 7/3

slope = 1/3

Solution of the equations

y = -x - 4   ----(1)

3y - x = -7  ----(2)

solve (1) and (2)

3(-x - 4) - x = -7

-3x -12 - x = -7

-4x = 5

x = -5/4

y = -5/4 - 4

y = -5.25

Thus, the system has one solution and the system consists of parallel lines.

Learn more about parallel lines here: https://brainly.com/question/24607467
#SPJ1

Find two functions defined
implicity by this equation:
(x − 5)² − 3(y + 2)² = 4

Answers

Answer:

[tex] \frac{dy}{dy} = \frac{x - 5}{3y + 6} [/tex]

Assume the random variable X has a binomial distribution with the given probability of obtaining a success. Find the following probability, given the number of trials and the probability of obtaining a success. Round your answer to four decimal places.

P ( X > 2 ) , n = 5 , p = 0.7

Answers

The value of the probability P(x > 2) is 0.8369

How to evaluate the probability?

The given parameters are:

n = 5

p =0.7

The probability is calculated as:

[tex]P(x) = ^nC_x *p^x * (1 - p)^x[/tex]

Using the complement rule, we have:

P(x > 2) = 1 - P(0) - P(1) - P(2)

Where:

[tex]P(0) = ^5C_0 *0.7^0 * (1 - 0.7)^5[/tex]

P(0) = 1 *1 * (1 - 0.7)^5 = 0.00243

[tex]P(1) = ^5C_1 *0.7^1 * (1 - 0.7)^4[/tex]

P(1) = 5 *0.7^1 * (1 - 0.7)^4 = 0.02835

[tex]P(2) = ^5C_2 *0.7^2 * (1 - 0.7)^3[/tex]

P(2) = 10 *0.7^2 * (1 - 0.7)^3 = 0.1323

Recall that:

P(x > 2) = 1 - P(0) - P(1) - P(2)

So, we have:

P(x > 2) = 1 - 0.00243 - 0.02835 - 0.1323

Evaluate

P(x > 2) = 0.83692

Approximate

P(x > 2) = 0.8369

Hence, the value of the probability P(x > 2) is 0.8369

Read more about probability at:

https://brainly.com/question/25870256

#SPJ1

Which of the following is NOT a rational expression?

Answers

The expression that is not rational is the first one:

[tex]f(x) = \frac{6x}{4}[/tex]

Which of the given expressions is not rational?

A rational expression is something of the form:

[tex]f(x) = \frac{q(x)}{p(x)}[/tex]

Such that q(x) can be any polynomial, and p(x) is a polynomial of at least degree 1.

This means that we need to have the variable "x" on the denominator.

Then is easy to recognize the expression that is not rational, is the one that does not have x on the denominator, which is the first one:

[tex]f(x) = \frac{6x}{4}[/tex]

If you want to learn more about rational expressions:

https://brainly.com/question/1851758

#SPJ1

Answer: B

Step-by-step explanation:

I took the test and this was the correct answer

What is the solution to t - 3 < 12

Answers

Answer:

[tex]\boxed {t < 15}[/tex]

Step-by-step explanation:

Given :

⇒ t - 3 < 12

Add 3 to each side :

⇒ t - 3 + 3 < 12 + 3

t < 15

Answer: t < 15

Step-by-step explanation:  

rearrange the terms                                       t - 3 < 12

calculate the sum or difference                    t < 12 + 3

answer                                                             t < 15                            

Need help ASAP, will mark brainliest if you can check my answer:

Answers

According to the compound interest model, we find the following results: I) x ≈ 11.5 yr, C' = $ 101317.36, II) r ≈ 7.4 %, x ≈ 9.8 yr, III) C = $ 7626.38, x ≈ 8.6 yr, IV) r ≈ 6.5 %, C = $ 12801.61

How to determined all the variables associated with compound interest

Compound interest describes the capital gain in term of deposited capital and the consideration that such capital is increased continuously in time. The compound interest model is shown below:

C' = C · (1 + r/100)ˣ     (1)

Where:

C - Initial capitalC' - Current capital r - Interest rate, in percentage.t - Time, in years

The doubling time (x) is the period needed for a capital to be doubled. It is described by the following expression based on (1):

x = (㏒ 2)/[㏒ (1 + r/100)]     (2)

Now we proceed to calculate each missing variable:

Case I - Doubling time

x = (㏒ 2)/[㏒ (1 + 6.2/100)]

x ≈ 11.5

Case I - Current capital

C' = 75000 · (1 + 6.2/100)⁵

C' = 101317.36

Case II - Interest rate

[tex]r = 100\cdot \left(\sqrt [5] {\frac{7130.90}{5000} }-1\right)[/tex]

r ≈ 7.4

Case II - Doubling time

x = (㏒ 2)/[㏒ (1 + 7.3/100)]

x ≈ 9.8

Case III - Initial capital

C = 11414.71/(1 + 8.4/100)⁵

C = 7626.38

Case III - Doubling time

x = (㏒ 2)/[㏒ (1 + 8.4/100)]

x ≈ 8.6

Case IV - Interest rate

[tex]r = 100\cdot \left(\sqrt [11] {2 }-1\right)[/tex]

r ≈ 6.5

Case IV - Initial capital

C = 17539.32/(1 + 6.5/100)⁵

C = 12801.61

To learn more on compound interest: https://brainly.com/question/14295570

#SPJ1

What is the partial fraction decomposition of StartFraction 7 x squared + 14 Over (x squared + 3) squared EndFraction?

Answers

The final decomposition of the given partial fraction is; 7 + (-7/(x² + 3))

How to decompose partial fractions?

We are given the polynomial fraction (7x² + 14)/(x² + 3) to decompose.

Now, due to the fact that the degree of the numerator is not less than the degree of the denominator, we will perform polynomial long division to get;

(7x² + 14)/(x² + 3) = 7 + (-7/(x² + 3))

Now, the second term (-7/(x² + 3)) cannot be decomposed further and as such, we say that our final decomposition of the given partial fraction is;

7 + (-7/(x² + 3))

Read more about partial fractions at; https://brainly.com/question/24236946

#SPJ1

Find the solution to the following system by substitution.
5x + y = 20
y = 5x

A. (2,10)
B. (4,0)
C. (0,20)
D. (4,20)​

Answers

Answer:

A. (2,10)

Step-by-step explanation:

5x + y = 20

y = 5x

The second equation is already solved for y, so substitute 5x for y in the first equation to find x.

5x + 5x = 20

10x = 20

x = 2

Now substitute 2 for x in the second original equation to find y.

y = 5(2)

y = 10

Answer: (2, 10) which is A.

Answer: A. (2, 10)

Step-by-Step Explanation:

=> 5x + y = 20 (Eq. 1)
=> y = 5x (Eq. 2)

Substitute value of ‘y’ from Eq. 2 in 1 :-

5x + y = 20
5x + (5x) = 20
10x = 20
x = 20/10
=> x = 2

Therefore, x = 2

Substitute value of ‘x’ in Eq. 2 :-

y = 5x
y = 5(2)
=> y = 10

Therefore, y = 10

Hence, x = 2 and y = 10 (2, 10)

Elian's credit card had a balance of $132.20 on April 1. On April 5, he charged $74.50. On April 18, he made a payment of $52. On April 22, hecharged $18.75 and did not use the credit card the rest of the month. What was his average daily balance? $166.76 $173.45 $206.70 $179.86

Answers

Answer:

I iitiriririrrifjjrjr idiocy jdkthink 179.86

You are given that cos(A)=−35, with A in Quadrant II, and cos(B)=817, with B in Quadrant I. Find cos(A−B). Give your answer as a fraction.

Answers

Expand cos(A - B) with the identity

cos(A - B) = cos(A) cos(B) + sin(A) sin(B)

A is in quadrant II, so sin(A) > 0, and B is in quadrant I, so sin(B) > 0. Using the Pythagorean identity, we get

cos²(A) + sin²(A) = 1   ⇒   sin(A) = + √(1 - (-3/5)²) = 4/5

cos²(B) + sin²(B) = 1   ⇒   sin(A) = + √(1 - (8/17)²) = 15/17

Then

cos(A - B) = (-3/5) × 8/17 + 4/5 × 15/17 = 36/85

cos (A - B) is 36/85

How to simply the identity

Expand cos(A - B) with the identity

You get, cos(A - B) = cos(A) cos(B) + sin(A) sin(B)

Since A is in quadrant II, so sin(A) > 0,

B is in quadrant I, so sin(B) > 0.

Using the Pythagorean identity, we get

cos²(A) + sin²(A) = 1  

Make sin A the subject of formula

[tex]sin(A)^{2}[/tex] =  ([tex]\sqrt{(1 - (-3/5}[/tex])²)

Find the square root of both sides, square root cancels square

[tex]sin A[/tex] = 4/5

Repeat the same for the second value

[tex]sin A^{2} = \sqrt{(1- 8/17)^2}[/tex]

[tex]sin A[/tex] = 15/17

Substitute values into cos(A - B)

cos(A - B) =  cos(A) cos(B) + sin(A) sin(B) = (-3/5) * 8/17 + 4/5 * 15/17

cos (A - B) = 36/85

Therefore, cos (A - B) is 36/85

Learn more about trigonometric identities here:

https://brainly.com/question/7331447

#SPJ1

Given: a || b, transversal k
Prove <3 = <6

Answers

By the property of corresponding angles  ∠3 ≅ ∠5.

What is transversal line?

In geometry, a transversal is any line that intersects two straight lines at distinct points.

If two parallel lines are cut by a transversal, then each pair of corresponding angles are equals.

That is, ∠3 = ∠5 and ∠4 = ∠6.

Therefore the given angles ∠3 and ∠5 are equal.

Hence  ∠3 ≅ ∠5 .

Know more about transversal line here :

https://brainly.com/question/17432060

#SPJ1

What is the value of x for the regular polygon shown below?
2x + 4
36

Answers

im not sure what your asking add more please

At the end of 2​ years, P dollars invested at an interest rate r compounded annually increases to an​ amount, A​ dollars, given by the following formula. Upper A equals Upper P (1 plus r )squared Find the interest rate if ​$32 increased to ​$50 in 2 years. Write your answer as a percent.

Answers

The interest rate will be equal to 24% in 2 years.

What is compound interest?

Compound interest is the interest levied on the interest. The formula for the calculation of compound interest is given as:-

Given that:-

Find the interest rate if ​$32 increased to ​$50 in 2 years.

The interest rate will be calculated by using the following formula:-

[tex]A = P[1+\dfrac{r}{n}]^{nt}[/tex]

[tex]50=32[1+\dfrac{r}{1}]^{2}[/tex]

[tex]\dfrac{50}{32}=(1+r)^2[/tex]

1.56 = ( 1 + r )²

√1.56 = ( 1 + r )

r = 1.24 - 1

r = 0.24

r = 24%

Therefore  interest rate will be equal to 24% in 2 years.

To know more about Compound interest follow

https://brainly.com/question/24924853

#SPJ1

when finding the margin of error of the mean of a normally distributed population from a sample, what is the critical probability, assuming a confindence level of 58%

Answers

The critical probability, assuming a confindence level of 58% is 0.79.

How to calculate probability?

From the information given, the confidence level is 58%. The alpha will be:

= 1 - (58/100)

= 1 - 0.58

= 0.42

The critical probability will be:

= 1 - (0.42/2)

= 1 - 0.21

= 0.79

Learn more about probability on:

brainly.com/question/19811911

#SPJ1

Write 1 3/5 as an improper fraction?

Answers

1 3/5 as an improper fraction is 8/5
13/5 is an improper fraction is 8/5

Diseases tend to spread according to the exponential growth model. In the early days of AIDS, the growth factor (i.e. common ratio; growth multiplier) was around 2.0. In 1983, about 1800 people in the U.S. died of AIDS. If the trend had continued unchecked, how many people would have died from AIDS in 2003?

Answers

Answer:

1,887,436,800

Step-by-step explanation:

f(t) = a·b^t

f (t) = number of cases at year t

a = starting value = 1800 in 1983

b = growth factor = 2

t = years since 1983 = 2003 - 1983 = 20

f(t) = 1800·(2)^20 =

1,887,436,800

wyzant

philip p

Two buses leave a station at the same time and travel in opposite directions. One bus travels 14 miles faster than the other. If the two buses are 640 miles apart after 5 hours, what is the rate of each bus?

Answers

If the two buses are 640 miles apart after 5 hours. Then the rate of each bus will be 57 miles per hour and 71 miles per hour.

What is speed?

The distance covered by the particle or the body in an hour is called speed. It is a scalar quantity. It is the ratio of distance to time.

We know that the speed formula

Speed = Distance/Time

Two buses leave a station at the same time and travel in opposite directions.

One bus travels 14 miles faster than the other.

Let x be the speed of first bus. Then the speed of the second bus will be (x + 14).

If the two buses are 640 miles apart after 5 hours.

Then the rate of each bus will be

Then the relative speed of the buses will be

S = x + x + 14

S = 2x + 14

Then the value of x will be

2x + 14 = 640 / 5

2x + 14 = 128

      2x = 114

        x = 57 miles per hour

Then the speed of the other bus will be

⇒ 57 + 14

⇒ 71 miles per hour

More about the speed link is given below.

https://brainly.com/question/7359669

#SPJ1

Design a real life activity for the Intermediate Phase in which learners will be required to apply the associative property of multiplication over addition.​

Answers

The associative property of adding or multiplying can be represented by (a + b) + c = a + (b + c) and a * (b * c) = (a * b) * c

What is an equation?

An equation is an expression that shows the relationship between two or more numbers and variables.

The associative property states that when adding or multiplying it can be represented by:

(a + b) + c = a + (b + c)

For multiplication:

a * (b * c) = (a * b) * c

Find out more on equation at: https://brainly.com/question/2972832

#SPJ1

Which statements about the system are true? Select two options.
y=-x-4
3y-x = -7
The system has one solution.
The system consists of parallel lines.
Both lines have the same slope.
Both lines have the same y-intercept.
The equations represent the same line.

Answers

The slope for both the line is m= 1/3 and  they are parallel lines , Option B and C are correct two options

The first equation is

 y = (1/3)x-4 and not y = -x-4

(if the equation is not corrected then it will not have two true statements)

What is a System of Equation ?

A system of equation is a set of equation which have a common solution

The given system of equation is

y = (1/3)x-4

3y -x = -7

3y = x-7

As it can seen from the standard equation of a line that

y =mx+c

so slope for both the line is m= 1/3

Therefore they are parallel lines

Thus , Option B and C are correct two options

To know more about System of Equation

https://brainly.com/question/12895249

#SPJ1

Are the two triangles congruent?

Answers

Answer:

yes.......................

Step-by-step explanation:


Which linear equation has a slope of 3 and a y-intercept of -2?
y = 3x + 2
y = 3x - 2
y = -2x + 3
y=-2x-3

Answers

Answer:

y = 3x - 2

Step-by-step explanation:

The number beside the x is the slope. The number at the end of the equation is the y-intercept.

All the answers are in this form:

y = mx + b

m is the slope.

b is the y-intercept.

With a 3 filled in for slope and -2 filled in for the y-intercept, you get:

y = 3x + -2

is the same as,

y = 3x - 2

Step 2 of 2 : Suppose a sample of 782 suspected criminals is drawn. Of these people, 548 were not captured. Using the data, construct the 98% confidence interval for the population proportion of people who are captured after appearing on the 10 Most Wanted list.

Answers

98% of confidence intervals for the Population proportion of people who captured after appearing on the 10 most wanted list

(0.6625, 0.7388).

What is confidence interval?

A confidence interval is the mean of your estimate plus and minus the variation in that estimate.

Given sample, n= 782

As, 548 were not captured.

x= 548

So, p= 548/ 782

     p = 0.7007

q= 1- 0.7007 = 0.2993

98% of confidence intervals for the Population proportion of people who captured after appearing on the 10 most wanted list

Now,

α = 1 - 0.98 = 0.02

α/2 = 0.02/2 = 0.01

For confidence level of 98% , z is 2.33

Now,

(0.70007 - 2.33√ (0.7007(0.2993))/782 , 0.70007 + 2.33 √(0.7007(0.2993))/782

= (0.6625, 0.7388)

Hence, 98% of confidence intervals for the Population proportion of people who captured after appearing on the 10 most wanted list

(0.6625, 0.7388).

Learn more about this concept here:

https://brainly.com/question/16519257

#SPJ1

Other Questions
Which region is richest in minerals such as copper, zinc, and silver?A. 2B. 1C. 3D. 4 After the execution of the following statement, the variable sold will reference the numeric literal value as (n) ________ data type. sold = 256.752 The underlined sentence is- A. a piece of evidence for the texts argument B. The thesis statement for the textC. A piece of reasoning in the textD. The conclusion of the text __________________________It connects ideas in the text by- A. Relating a fact or story that demonstrates the texts argument B. Making the main claim of the textC. Suggesting how a specific example relates to the broader argument D. Describing why the information provided matters to the readers If the mortgage loan is 80% of the appraised value of a house, and the interest rate of 8% amounts to $460 interest for the first month, then what is the appraised value of the house? Monomal by a DinormalWhat is the product of x(x + 1)?O 2x+x0x2+2x022 +1O x + xwill give brainliest Of protons, neutrons, and electrons, the least massive particle is the electron.is it True or False Hey there - Describe about French revolution. On a coordinate plane, 2 exponential functions are shown. f (x) decreases from quadrant 2 to quadrant 1 and approaches y = 0. It crosses the y-axis at (0, 4) and goes through (1, 2). g (x) increases from quadrant 3 into quadrant 4 and approaches y = 0. It crosses the y-axis at (0, negative 4) and goes through (1, negative 2).Which function represents g(x), a reflection of f(x) = 4(one-half) Superscript x across the x-axis?g(x) = 4(2)xg(x) = 4(2)xg(x) = 4(one-half) Superscript xg(x) = 4(one-half) Superscript negative x Hull fouling is a(n) _______ introduction of nonnative species. a. unintentional b. intentional c. unimportant d. land-based please select the best answer from the choices provided a b c d Find the measure of x.42X17x= [ ?Round to the nearest hundredth. What is the measure of an exterior angle of a regular hexagon?3601803060 The population of a large city can be calculated using the function P=345,000(1.01). What can you say about the rate of change from year 1 to year 2 compared to the rate of change from year 9 to year 10? A medical assistant is answering the clinic phone. Which of the following guidelines demonstrates professional telephone etiquette? what causes the hidden node problem in a wireless local area network (wlan)? Which equations are true for x = -2 and x = 2? Select two optionsA x-4=0B x=-4B 3x + 12 = 0C 4x = 16D 2(x-2) = 0 If you receive a 30% DISCOUNT off an item costing $60.00, how much money do you save? A government authority or license excluding others from making, using, orselling an invention without the creator's permission is called aO A. patentOB. Copyright O c. trademarkOD. trade secret giving 20 points out for this question! After the aed delivers a shock, what is your next step? begin cpr compressions deliver another shock do nothing remove the aed pads Solve for x.Use the quadratic formula.2x - 9x + 8 =0